When people evade income taxes by not declaring

This topic has expert replies
Legendary Member
Posts: 784
Joined: Sun Apr 03, 2011 3:51 am
Thanked: 114 times
Followed by:12 members
When people evade income taxes by not declaring taxable income, a vicious cycle results. Tax evasion forces lawmakers to raise income tax rates, which causes the tax burden on nonevading taxpayers to become heavier. This, in turn, encourages even more taxpayers to evade income taxes by hiding taxable income. The vicious cycle described above could not result unless which of the following were true?

(A) An increase in tax rates tends to function as an incentive for taxpayers to try to increase their pretax incomes.
(B) Some methods for detecting tax evaders, and thus recovering some tax revenue lost through evasion, bring in more than they cost, but their success rate varies from year to year.
(C) When lawmakers establish income tax rates in order to generate a certain level of revenue, they do not allow adequately for revenue that will be lost through evasion.
(D) No one who routinely hides some taxable income can be induced by a lowering of tax rates to stop hiding such income unless fines of evaders are raised at the same time.
(E) Taxpayers do not differ from each other with respect to the rate of taxation that will cause them to evade taxes.

What type of CR is this? Is this assumption?

Senior | Next Rank: 100 Posts
Posts: 62
Joined: Fri Aug 12, 2011 12:37 am
Thanked: 3 times
Followed by:1 members

by sk8legend408 » Sun Dec 18, 2011 5:03 am
I think it is an assumption CR. The way the passage seems to be structured is through a causal theme as the passage states that tax evasion causes lawmakers to increase taxes which causes taxpayers to evade income taxes by hiding taxable income.

I believe the answer is E as this cycle can only result if more tax payers try to hide taxable income. E shows that tax payers do not differ from each other and increases in tax rates will cause them to evade taxes.

User avatar
GMAT Instructor
Posts: 15539
Joined: Tue May 25, 2010 12:04 pm
Location: New York, NY
Thanked: 13060 times
Followed by:1906 members
GMAT Score:790

by GMATGuruNY » Sun Dec 18, 2011 6:07 am
patanjali.purpose wrote:When people evade income taxes by not declaring taxable income, a vicious cycle results. Tax evasion forces lawmakers to raise income tax rates, which causes the tax burden on nonevading taxpayers to become heavier. This, in turn, encourages even more taxpayers to evade income taxes by hiding taxable income. The vicious cycle described above could not result unless which of the following were true?

(A) An increase in tax rates tends to function as an incentive for taxpayers to try to increase their pretax incomes.
(B) Some methods for detecting tax evaders, and thus recovering some tax revenue lost through evasion, bring in more than they cost, but their success rate varies from year to year.
(C) When lawmakers establish income tax rates in order to generate a certain level of revenue, they do not allow adequately for revenue that will be lost through evasion.
(D) No one who routinely hides some taxable income can be induced by a lowering of tax rates to stop hiding such income unless fines of evaders are raised at the same time.
(E) Taxpayers do not differ from each other with respect to the rate of taxation that will cause them to evade taxes.

What type of CR is this? Is this assumption?
The question asks for WHAT MUST BE TRUE -- in other words, for the ASSUMPTION.

Premises are FACTS. Only one fact is given in the passage: When people evade income taxes.
The rest of the passage offers PREDICTIONS. Predictions are not facts.

The first connection made by the passage involves a language shift.
The premise is about X: People EVADE income taxes.
The conclusion is about Y: Lawmakers WILL RAISE income tax rates.

The passage assumes that X is linked to Y: that if people EVADE income taxes, lawmakers WILL RAISE income tax rates.

What must be true for this connection to be valid?
Answer choice C: IT MUST BE TRUE that when lawmakers establish income tax rates, they do not allow adequately for revenue that will be lost through evasion.
C provides the necessary ASSUMPTION: in order to conclude that the lawmakers will RAISE income tax rates, it must be true that -- when the lawmakers establish the initial rates -- they do not allow adequately for the revenue that will be lost through EVASION.

The correct answer is C.
Private tutor exclusively for the GMAT and GRE, with over 20 years of experience.
Followed here and elsewhere by over 1900 test-takers.
I have worked with students based in the US, Australia, Taiwan, China, Tajikistan, Kuwait, Saudi Arabia -- a long list of countries.
My students have been admitted to HBS, CBS, Tuck, Yale, Stern, Fuqua -- a long list of top programs.

As a tutor, I don't simply teach you how I would approach problems.
I unlock the best way for YOU to solve problems.

For more information, please email me (Mitch Hunt) at [email protected].
Student Review #1
Student Review #2
Student Review #3

Senior | Next Rank: 100 Posts
Posts: 78
Joined: Wed May 16, 2012 8:57 pm
Thanked: 2 times

by divineacclivity » Sat Nov 03, 2012 4:35 am
GMATGuruNY wrote: The question asks for WHAT MUST BE TRUE -- in other words, for the ASSUMPTION.

Premises are FACTS. Only one fact is given in the passage: When people evade income taxes.
The rest of the passage offers PREDICTIONS. Predictions are not facts.

The first connection made by the passage involves a language shift.
The premise is about X: People EVADE income taxes.
The conclusion is about Y: Lawmakers WILL RAISE income tax rates.

The passage assumes that X is linked to Y: that if people EVADE income taxes, lawmakers WILL RAISE income tax rates.

What must be true for this connection to be valid?
Answer choice C: IT MUST BE TRUE that when lawmakers establish income tax rates, they do not allow adequately for revenue that will be lost through evasion.
C provides the necessary ASSUMPTION: in order to conclude that the lawmakers will RAISE income tax rates, it must be true that -- when the lawmakers establish the initial rates -- they do not allow adequately for the revenue that will be lost through EVASION.

The correct answer is C.
And we did NOT pick option D because D would rather tell how we could lower the evasion rate whereas the question is regarding the vicious cycle of evading and raising which is best answered/broken by taking evasion losses into account while planning itself so that the taxes do not have to be raised as raising causes further more evasion & evasion causes more raising & hence the cycle. D doesn't stop the cycle, rather nips in the bud by controlling evasion at first - also, avoiding evasion absolute might be a far fetched dream or too theoretical. So, even if it is small evasion (by deploying controlling methods), the cycle might still occur if loses due to evasion are not taken into account in the beginning :)
Is my though process up to the mark? Thanks in advance.